You are on page 1of 51

ELPT SHORT COURSE

GENERAL STRATEGIES
1. Use your time wisely. The ELPT is a timed test. You must work very quickly and efficiently to
finish all of the questions in the time you are allowed. The skill strategy practice taught in this
course will help you learn to budget your time on each section of the ELPT test. In addition,
you should bring a watch to the test so that you can keep track of your time.

2. Don’t read the instructions to each ELPT test section. The test instructions used in exercises
during the course are exactly the same as the ones used on the ELPT test. You should
become familiar with these instructions. If you are familiar with them before you take the ELPT
test, you do not have to read them when you begin the test. Instead, when permitted, you can
move immediately to the test questions and begin working.

3. If you do not know the answer to a question, GUESS. This is a very important strategy to use
when taking the ELPT test. There is no penalty for guessing. You have a twenty-five percent
chance of guessing the correct answer to each question. Unmarked answers will be
counted as wrong and will lower your score.

4. Mark your answer sheet very carefully. To prevent marking answers in the wrong order, you
should follow your place on your answer sheet with one finger. Check to see that the number
next to this finger is the same as the number of the question you are looking at in your test
booklet. When you choose your answer, fill in the circle completely. If you need to change
the answer, erase it completely, and mark your new answer.

5. Do not write in your test booklet. You are not allowed to make any marks on your ELPT test
booklet during the test.

6. The night before the test, relax. Do not try to do any serious studying the night before the test.
This will only make you nervous and tired. The night before the ELPT test, it is a good idea to
relax and go to bed early. Then, you will be at your best for the test.

7. It is a good idea to eat something substantial before the ELPT test. This test is a long exam,
and having something to eat beforehand can help you to focus and concentrate on the test.

8. Arrive at the test center ahead of time. If you are late for the ELPT test, you will not be
allowed to take it.

1
SCORE CONVERSION TABLE
Use the following table to estimate your ELPT scores

Number Correct (Cs) Converted Scores


Section 1 Section 2 Section 3
50 68 - 67
49 67 - 66
48 66 - 65
47 65 - 63
46 63 - 61
45 62 - 60
44 61 - 59
43 60 - 58
42 59 - 57
41 58 - 56
40 57 68 55
39 57 67 54
38 56 65 54
37 55 63 53
36 54 61 52
35 54 60 52
34 53 58 51
33 52 57 50
32 52 56 49
31 51 55 48
30 51 54 48
29 50 53 47
28 49 52 46
27 49 51 46
26 48 50 45
25 48 49 44
24 47 48 43
23 47 47 43
22 46 46 42
21 45 45 41
20 45 44 40
19 44 43 39
18 43 42 38
17 42 41 37
16 41 40 36
15 41 40 35
14 39 38 34
13 38 37 32
12 37 36 31
11 35 36 30
10 33 33 29
9 32 31 28
8 32 29 28
7 31 27 27
6 30 26 26
5 29 25 25
4 28 24 24
3 27 23 24
2 26 22 23
1 25 21 22
0 24 20 21

2
LISTENING COMPREHENSION
PART A QUESTION
SHORT CONVERSATION
For each of the thirty questions in part A of the Listening Section of the ELPT test, you will hear a short
conversation between two speakers followed by a question. The conversation and question are not
written in your test book. After you listen to the conversation and question, you must choose the best
answer to the question from your test book.

For Example :

On the recording, you hear:


(man) I need a car to drive to Arizona, and I don’t have one.
(woman) Why not rent one?
(narrator) What does the woman suggest?

In your test book , you read:

(A) Getting a red car.


(B) Renting an apartment in Arizona.
(C) Not driving to Arizona.
(D) Renting a car for the trip.

Answer (D) is the best answer to the question. Why not rent one? is a suggestion that he rent a car for
his trip to Arizona.

STRATEGIES FOR THE LISTENING PART A QUESTION

1. As you listen to each short conversation, focus on the second line of the conversation. The
answer to the question is generally found in the second line of the conversation.

2. Keep in mind that the correct answer is probably a restatement of a key word or idea in the
second line of the conversation. Think of possible restatements of the second line of the
conversation.

3. Keep in mind that certain structure and expression are tasted regularly in Listening Part A.
Listen for these structure and expression:
Restatement Negatives
Suggestion Passive
Conclusion about who and where Expression of agreement

4. Keep in mind that these questions generally progress from easy to difficult. This means that
question 1 through 5 will be the easiest question and question 26 through 30 will be the
hardest question.

5. Read the question and chose the best answer to each question. Remember to answer each
question even if you not sure of the correct response. Never leave any answer blank.

6. Even if you do not understand the complete conversation, you can find the correct answer.

3
RESTATEMENT
S
Often the correct answer in part A of the Listening Section is an answer that contains a restatement of
the ideas in the second line of the conversation.

Example
On the recording, you hear:

(women) Steve, is something the matter? You don’t look very good.
(man) Oh, I’m feeling a little sick today.
(narrator) What does the man mean?

In your test book, you read:

(A) He’s not very good-looking.


(B) He’s a bit ill.
(C) He looks worse than he feels.
(D) His feet are a little thick.

In this conversation, sick means ill, and a little means a bit. The best answer to this question is
therefore answer (B) because answer (B) restates the idea in the second line of the dialogue. Please
note that the answer (D) is definitely not a correct answer because it contains feet instead of feel and
thick instead of sick. These words are similar in sounds but not in meaning to the word that you hear
on the recording.

STRATEGIES
1. Focus on the second speaker’s line.

Sample Problem

On the recording you will hear:


(woman) What do you like about your new house?
(man) It’s very close to a park.
(narrator) What does the man mean?

In your test book you will read:


(A) The house is closed up now.
(B) He parks his car close to his house.
(C) His home is near a park.
(D) He doesn’t really like his new house.

The correct answer is C. Focus on the second speaker’s line. It’s very close to a park in the
second speaker’s statement means that the man likes his new house because it’s located near the
park.

2. Find a restatement of the key idea(s) on the second speaker’s line.

Sample problem

On the recording you will hear:


(woman) Sam, you’ve been working at the computer for hours.
(man) Yes, and I need to take a break.
(narrator) What does the man mean?

In your test book you will read:


(A) The computer’s broken.
(B) He needs to relax for while.
(C) He should keep working.
(D) He’s broke.

4
The correct answer is B. Find a restatement of the key idea(s) on the second speaker’s line. I need
to take a break means that the man needs to relax for a while. Relax for a while is the restatement
of take a break.

3. Avoid similar sounds.

Sample problem

On the recording you will hear:


(man) Mark said some really nice things to me.
(woman) He’s very grateful for what you did.
(narrator) What does the woman say about Mark?

In your test book you will read:


(A) He did a great job.
(B) He bought a crate full of fruit.
(C) He made a great fool of himself.
(D) He’s thankful.

The correct answer is D. Avoid similar sounds. Great job in answer A might sound similar to
grateful on the second speaker’s line. A crate full in answer B might sound similar to grateful on
the second speaker’s line. A great fool in answer C might sound similar to grateful on the second
speaker’s line. Thankful in answer D is a restatement of grateful on the second speaker’s line.

NEGATIVES
Negative expressions are very common in Listening Part A. the most common kind of correct
response to a negative statement is a positive statement containing a word with an opposite meaning.

Example

On the recording, you hear:


(Woman) Did you get a lot of work done at the library today?
(man) I couldn’t. It wasn’t very quiet there.
(narrator) What does the man mean?

In your test book, you read:

(A) The library was noisy.


(B) He got a lot done.
(C) He couldn’t quite get to the library today.
(D) The library is good place to work because it’s quite.

The correct answer is answer (A). It was not quite in the library, this means that it was noisy. Notice
that the correct answer uses noisy, the opposite of quiet.

Negative Expressions Example


not Tom is not sad about the result
- n’t happy

The door isn’t open


closed

Steve did not pass the class.


failed

5
STRATEGIES
1. Find the opposite meaning of the negative expressions

Sample problem

On the recording you will hear:

(woman) You made so many mistakes in this homework.


(man) I wasn’t very careful.
(narrator) What does the man mean?

In your test book you read:

(A) He was rather careless.


(B) He does not care about mistakes.
(C) He took care of the work at home.
(D) He did not carry the work home.

The correct answer is A. Find the opposite meaning of the negative expression. The negative
expression wasn’t very careful on the second speaker’s line means that the man was rather
careless. The word careless in answer A is the opposite of the word careful on the second
speaker’s line.

Another sample problem

On the recording you will hear:


(man) Is there a lot of soup? I’m kind of hungry.
(woman) Sorry, there’s not a lot.
(narrator) What does the woman mean?

In your test book you read:


(A) There’s not very much soap.
(B) She doesn’t like soup.
(C) There’s only a little soup.
(D) The man should not be hungry.

The correct answer is C. The sentence there’s not a lot on the second speaker’s line means that
there’s only a little soup. The word a little on answer C is the opposite meaning of the word a lot on
the second speaker’s line.

SUGGESTIONS
Suggestions are common in Listening Part A, so you should become familiar with them. The following
example shows an expression of a suggestion.

Example
On the recording, you hear:

(man) I haven’t talked with my parents in a while.


(woman) Why don’t you call them now?
(narrator) What does the woman suggest?

In your test book, you read:

(A) Calling off his visit.


(B) Talking about his parents.
(C) Calling his parents in a while.
(D) Phoning his family.

6
In this example, the expression why don’t is an expression of suggestion, so the woman’s suggestion
is to call them. This means that the woman suggest phoning his family. The best answer is therefore
answer(D).

The following chart lists common expressions that show suggestion.

Expression of suggestion

Why...not...?
Why not...?
Let’s...

PASSIVE

It is sometimes difficult to understand who or what is doing the action in a passive sentence. This
problem is often tested in Listening Part A.

Example

On the recording, you hear:

(man) Is that a new chair?


(woman) Yes, we just bought it last week.
(narrator) What does the woman mean?

In your test book you read:

(A) She brought the chair with her.


(B) The chair was lost for a week.
(C) The chair was purchased recently.
(D) She bought the last chair from the store.

In this conversation, the woman uses an active idea, we... bought it [the chair]. The correct answer
uses the passive idea the chair was purchased. Therefore, the best answer to the question above is
answer (C).

You should note the following about passive sentences in Listening Part A.

PASSIVE STATEMENTS

1. If the conversation contains a passive statement, the answer to the question is


often an active statement.
2. If the conversation contains an active statement, the answer to the question is
often a passive statement.

NOTE: Check carefully who or what is doing the action in these questions.

7
WHO AND WHERE

It is common in Listening Part A to ask you to draw some kind of conclusion. In this type of question
the answer is not clearly stated; instead you must draw a conclusion based on clues given in the
conversation. One kind of conclusion that is common in this part of the test is to ask you to determine
who the speaker is, based on clues given in the conversation.

Example
On the recording, you hear:

(man) What do you do during your performances?


(woman) I play the piano and sing.
(narrator) Who is the woman most likely to be?

In your test book, you read:

(A) An athlete.
(B) A member in a music store.
(C) A dancer.
(D) A musician.

The clues performances, piano, and sing in the conversation tell you that the woman is probably a
musician. Answer (D) is therefore the correct answer.
Another type of conclusion that is common in Listening Part A is to determine where the conversation
probably takes place, based on clues given in the conversation.

Example
On the recording, you hear:

(man) I’d like to open an account, please.


(woman) What type of account would you like, a checking or savings account?
(narrator)Where does this conversation probably take place?

In your test book, you read:

(A) In an accounting class.


(B) In a bank
(C) At service station.
(D) In a market.

The clue checking or saving account in the conversation tells you that this conversation probably takes
place in a bank. Answer (B) is therefore the correct answer.

The following chart outlines the key point that you should remember about this type of question.

CONCLUSIONS ABOUT WHO AND WHERE


It is common for you to be asked to draw the following conclusions in Listening Part A:
1 Who is probably talking?
2 Where does the conversation probably take place?

8
AGREEMENT
Expressions of agreement are common in Listening Part A, so you should become familiar with them.
The following example shows agreement with a positive statement.

Example

On the recording, you hear:


(man) I thought that the meal was overpriced.
(woman) Me, too.
(narrator) What does the woman mean?

In your test book, you read:


(A) There were too many spices in the meat.
(B) She has the same opinion of the meal as the man.
(C) She wants to share the man’s meal.
(D) The price of the meal was great.

The expression me, too shows agreement with a positive statement, so the woman means that she
has the same opinion as the man. The best answer is therefore answer (B).
The following chart lists common expressions that show agreement. You should become
familiar with these expressions.

EXPRESSIONS OF AGREEMENT

So do I. I’ll say. Me, too. You can say that again.

PART B & C: LONGER CONVERSATION


AND TALKS
Part B of the Listening Section of the ELPT test consists of two long conversations while in
Part C consicts of three talks. After each conversation and talk, there will be a number of questions.
You will hear the conversations, talks and the questions on the recording; they are not written in your
test book. You must choose the best answer to each question from the four choices that are written in
your test book.
The conversations are often about some aspect of school (how difficult a class is, how to write
a research paper, how to register for a course) or about general living (renting an apartment, playing
sports, and going to the bank). The conversations can also be about topics currently in the news in the
United States (desalination of the water supply, recycling of used products, and damage from a storm
or some other types of natural phenomena).

Example For Part B


On the recording, you hear:
(narrator) Questions 1 through 4. Listen to a conversation between two people who are decorating
an apartment.
(woman) Hey, Walt. Do you think you could help me hang these pictures on the wall?
There are only two of them.
(man) Sure, Monica. Where do you want them to go?
(woman) I’d like the picture of the mountains over the fireplace, and I’d like the picture of
my family over the sofa. What do you think?
(man) I think they’ll look fine there. How about if you hold the pictures while I hammer the
nails into the wall?
(woman) Okay. Let’s start with the picture of my family.

9
Questions:
1. On the recording, you hear:
(narrator) What are the man and woman discussing?

In your test book, you read:


(A) Taking some pictures.
(B) Hanging some plants.
(C) Taking a trip to the mountains.
(D) Putting some pictures on the wall.

2. On the recording, you hear:


(narrator) How many pictures are there?

In your test book, you read:


(A) One.
(B) Two.
(C) Three.
(D) Four.

3. On the recording, you hear:


(narrator) Where is the picture of the woman’s family going?

In your test book, you read:


(A) In the fireplace.
(B) Above the sofa.
(C) Home with Walt.
(D) To the top of the mountain.

4. On the recording, you hear:


(narrator) What is Walt probably going to do next?

In your test book, you read:


(A) Sit on the sofa.
(B) Photograph Monica’s family.
(C) Hammer the nails into the wall.
(D) Climb the walls.

The first question asks what the man and woman are discussing. Since the woman asks the man to
help hang these pictures on the wall, the best answer to this question is (D), putting some pictures on
the wall. The second question asks how many pictures there are, and the woman clearly says that
there are two, so the best answers is answers (B). The third question asks where the family picture is
going. The woman says that she would like the family picture over the sofa, so the best answer to this
question is (B), above the sofa. The last question asks what Walt is probably going to do. Walt has
suggested that he should hammer the nails into the wall, so the best answer is answer (C).

Example For Part C

On the recording, you hear:

(narrator) Questions 1 through 3.

Listen to a talk about cats.

(woman) Many people are allergic to cats. If they come in contact with cats, they sneeze, their
skin turns red, and their eyes begin to burn. However, it is not only people who suffer
from allergies. Cats may also be allergic to pollen, dust, and perfumes, many of the
same agents that cause allergies in people. Perhaps your cat is sneezing and has

10
watery eyes. If you think that your cat has some allergies, a veterinarian can prescribe
medication to help solve the problem.
On the recording, you hear :
(narrator) 1. What happens to people who suffer from allergies?
2. What is mentioned about cats?
3. What can someone do with a cat that has allergies?
In your test book, you read (same time):

1. (A) They shout.


(B) They drive red cars.
(C) They sneeze.
(D) They close their eyes.

2. (A) They often wear perfume.


(B) They can have allergies.
(C) They don’t ever suffer.
(D) They like dust and pollen.

3. (A) Nothing.
(B) Bathe it frequently.
(C) Put it outside.
(D) Give it medicine.

The first question asks what happens to people who suffer from allergies. The speaker says that they
sneeze, so the best answer is answer (C). The second question asks what is mentioned about cats.
The speaker says that cats may also be allergic, so the best answer is answer (B), they can have
allergies. The third question asks what to do with a cat that has allergies. The speaker says that a
veterinarian can prescribe medication, so the best answer is answer (D), give it medicine.

STRATEGIES FOR THE LISTENING PART B and PART C QUESTIONS


1. If you have the time, preview the answers to the Listening Part B questions. While
you are looking at the answers, you should try to anticipate the questions for each of the
groups of answers.
2. Listen carefully to the first line of the conversation. The first line of the conversation
often contains the main idea, subject, or topic of the conversation, and you will often be
asked to answer such questions.
3. As you listen to the conversation, follow along with the answers in your test book,
and try to determine the correct answers. Detail questions are generally answered in
order in the conversation, and the answers often sound the same as what is said on the
recording.
4. You should guess even if you are not sure. Never leave any answers blank.
5. Use any remaining time to look ahead at the answers to the questions that follow.

11
STRUCTURE AND WRITTEN EXPRESSION

1. SUBJECTS, OBJECTS AND NOUN COMPLEMENTS

All of the English sentences tested in Section Two of the ELPT contain subjects. In addition, many of
them contain objects (direct, indirect, or objects of prepositions) and noun complements.
The subjects, objects, and noun complements of English sentences normally occur as noun (phrase),
pronoun, gerund (phrase), infinitive (phrase), or noun clause.

Subject: Doer of the action in a sentence

Example: Andi has breakfast at 07.00 in the morning.

Object:
1. Direct receiver of the action of a sentence (DO)
2. Indirect receiver of the action of a sentence (IO)
3. Object of a preposition (O of Prep)

Example: I gave Andrew a CD for his birthday


IO DO O of Prep
I gave a CD to Andrew for his birthday
DO O of Prep O of Prep

Noun Complement:
1. Subject identifier—after the verb BE (SC)
2. Object identifier—after the direct object of a sentence (OC)

Example: Drew is a teacher.


SC
They elected him president
OC
The subjects, objects, and noun complements of English sentences normally occur as one of the five
noun structure forms given below.

Noun (phrase): Bali is a beautiful island.


S SC
Pronoun: We love Indonesia.
S
I taught them mathematics last semester.
S DO
Gerund (phrase): Reading books is my favorite hobby.
S
I relax by reading books.
O of Prep
Infinitive (phrase): We like to read books.
DO
Noun Clause: Everyone says that you like reading books
DO

Model test:
They didn’t know ___________________
(A) What to do
(B) Do
(C) To do what
(D) They should do

What is needed in this sentence? The direct object


Explanation: In this question, the direct object is being tested. Answer (B) is a verb. It cannot be a
direct object. In answer (C), the word order of the direct object is incorrect. In answer (D), the direct
object is incomplete. Answer (A) is the correct answer to the question because it contains all the parts
of a direct object in the correct order.

12
2. SUBJECTS AND VERBS

The most common types of problems in the structure section of ELPT test have to do with subjects
and verbs. The sentence may miss a subject, a verb, or both a subject and a verb.

Model test:

_____ was backed up for miles on the freeway.

(A) At the beach


(B) In the morning
(C) Traffic
(D) Cars

Explanation: In this example, there is a verb (was), but there is no subject. Answer (C) is the best
answer because it is a singular subject that agrees with the singular verb was.

3. EXPRESSIONS OF QUANTITY

Some English expressions of quantity are used only with countable nouns (nouns which can be made
plural, e.g. boy – boys). Other expressions of quantity are used only with uncountable nouns (nouns
that normally cannot be made plural, e.g. happiness. Still other expressions of quantity are used with
both countable and uncountable nouns.

USED WITH COUNTABLE NOUNS USED WITH UNCOUNTABLE NOUNS

many much
number of amount of
few little
a few a little
fewer less
none none
some some
any any
a lot of a lot of
one, two, three,….
Several

Model test:

The professor will require us to conduct research, many of which can be done in our own library.
A B C D

What is wrong with this sentence? Many and research are not in agreement.

Explanation: In this question research is an uncountable noun. The expression of quantity, many,
refers back to research, but many cannot be used with uncountable nouns. Answer (C) is the correct
answer to the question because many is incorrect.
Correction: much

4. ARTICLES
The English articles are a/an and the. The article a/an is used with singular, countable indefinite nouns
(nouns referring to things that are new to either the speaker or the listener). The spelling of the article
changes from a to an when the word it precedes begins with a vowel sound.

I need to buy a book.


I need to buy an English book.

13
The article the is used with singular and plural nouns as well as uncountable nouns. However, the
article, the only occurs with definite nouns (nouns referring to things the speaker and listener already
know about or which are made specific by their use in a sentence.)

The book I wanted is The ELPT Preparation Course.

Model test

1. Patricia is taking a very difficult classes this semester.


A B C D
What is wrong with this sentence? This article and the noun it refers to do not agree.

Explanation: In this question, a singular article is being used with a plural noun. The article a should
only be used with singular, indefinite, countable nouns. In this sentence, a is used with the noun
classes which is plural. Answer (C) is the correct answer because classes is the incorrect form of the
noun.
Correction: class

2. The mongoose is very clever creature that catches snakes.


A B C D

What is wrong with this sentence? An article is missing.

Explanation: In this question, the singular indefinite noun creature needs an article. Answer (B) is the
correct answer to the question because is very clever is incorrect.
Correction: is a very clear

3. A sun was shining brightly when we arrived at the lake.


A B C D

What is wrong with this sentence? An incorrect article has been used.

Explanation: In this question the wrong article has been chosen. Sun is a definite noun. There is only
one sun above the earth; it is definite to everyone. Definite nouns do not take the article a. Answer (A)
is the correct answer to this question because a sun is incorrect.
Correction: the sun

5. PERSONAL, POSSESSIVE, AND REFLEXIVE PRONOUNS

In English, pronouns are used to replace or refer to nouns, gerunds, infinitives, and sometimes entire
clauses. Pronouns change form depending on their functions in sentences.

PRONOUN

Personal Possessive
Reflexive
Subject Object Adjective Pronoun

I me my mine myself
You you your yours yourself
He him him his himself
She her her hers herself
It it its ------ itself
We us our ours ourselves
They them their theirs themselves
One one one’s ------- oneself

14
Model test

Although Alexander liked him new apartment very much, he found it somewhat noisy.
A B C D
What is wrong with this sentence? The possessive adjective form

Explanation: In this question, the pronoun him is in object form. It should be in possessive adjective
form. Answer (A) is the correct answer to this question because him is incorrect.
Correction: his

6. ADJECTIVES AND NOUNS

English nouns are often modified by adjectives. Adjectives usually come before the nouns they modify.
Adjectives cannot be made plural. Nouns must be made plural when they are countable and plural in
meaning.

Model test:

1. A bolt of lightning travels twenty thousands miles in one second.


A B C D

What is wrong with this sentence? The form of an adjective

Explanation: In this question, twenty thousands is a compound adjective modifying miles. As


discussed above, adjectives cannot be made plural in English. Answer (B) is the correct answer to this
question because twenty thousands is incorrect.
Correction: twenty thousand

2. Some trees grow to be over three hundred foot high.


A B C D

What is wrong with this sentence? The form of a noun

Explanation: In this question, three hundred is a compound adjective. It is plural in meaning, and the
nun it modifies should be plural. Foot is not in its plural form in this sentence. Answer (D) is the correct
answer to this question.
Correction: feet

3. The paint bright yellow that she chose for her bedroom creates a cheery atmosphere.
A B C D

What is wrong with this sentence? The order of the adjectives

Explanation: In this question, the adjectives bright and yellow come after the noun paint, which they
modify. They should come before this noun. Answer (A) is the correct answer to this question because
paint bright yellow is incorrect.
Correction: bright yellow paint

7. VERB TENSE FORM

In English, only two tenses are marked in the verb alone, present (as in "he sings") and past (as in "he
sang"). Other English language tenses, are marked by other words called auxiliaries.

The basic form (or root of the verb is the form listed in the dictionary and is usually identical to the
first person singular form of the simple present tense (except in the case of the verb "to be"):walk,
paint, think, grow, and sing

The past form of verbs is a little trickier. If the verb is regular (you can create the past form by adding
"-ed", "-d", or "-t" to the present form. When a basic form ends in "-y", you changed the "-y" to "-i-"; in
many cases you should also double terminal consonants before adding "-ed": walked, painted,
thought, grew, and sang).

15
There are three most common problematic situations with verbs:

 Check what comes after have


Whenever you see the verb have in any of its forms (have, has, having, had), be sure that the
verb that follows it is in the past participle form.

Have + Past Participle

For example:
They had walk* to school. (should be had walked)
We have see* the show (should be have seen)
He has took the test (should be has taken)
Having ate*, he went to school (should be having eaten)
She should have did* the work (should be should have done)

 Check what comes after be


The verb be in any of its forms (am, is, are, was, were, be, been, being) can be followed by
another verb. This verb should be in the present participle or the past participle.

BE + (1) Present Participle


(2) Past Participle

For example:
We are do* our homework (should be are doing)
The homework was do* early (should be was done)
Tom is take* the book (should be is taking)
The book was take* by Tom (should be was taken)

 Check what comes after will, would, and other modals.


Whenever you see a modal, such as will, would, shall, should, can, could, may, might, or
must, you should be sure that the verb that follows it is in its base form.

MODAL + Main form of the Verb

For example:
The boat will leaving* at 3.00 (should be will leave)
The doctor may arrives* soon (should be may arrive)
The students must taken* the exam (should be must tak)

Model test :
1. The assignment was did by most of the class
A B C D
What is wrong with this sentence? The formation of the passive

Explanation: In this question, the passive is incorrectly formed. The past form of do has been used
when the past participle is needed. Answer (B) is the correct answer to the question because was did
is incorrect.
Correction: was done

2. The people of this country have been expressed great concern about the environment
A B C D
What is wrong with this sentence? The passive has been used when the active is needed

16
Explanation: In this question, the passive form of the verb is used. However, the sentence contains a
direct object, great concern. Direct objects come after active verbs, not passive verbs. Answer (C) is
the correct answer to this question because has been expressed is incorrect.
Correction: have expressed

3. The books and toys ____________ to the orphanage.


(A) being donated
(B) that was donated
(C) donated
(D) was donated

What is needed in this sentence? The passive form.


Explanation: In this question, the passive form of the verb is needed. Answer (A) does not contain a
finite verb. Answer (B) contains more than a finite verb. Answer (C) looks like an active finite verb.
However, the books and toys cannot be the subject of donate. A piano cannot donate things. Only
humans can donate. Answer (D), which contains a correctly formed passive verb, indicates that
someone donated the piano. Using this passive verb form, the piano becomes the receiver, not the
doer, of the action. Answer (D) is the correct answer to this question.

8. VERB TENSE MEANING

Simple Present They walk


Present Perfect They have walked
Simple Past They walked
Past Perfect They had walked
Future They will walk
Future Perfect They will have walked

The meaning of a tense used in an English sentence must agree with the time meaning of the rest of
the sentence. The time meaning of a sentence is often determined by word or expressions that act as
time markers. Some of the most common English time markers are listed below.
Simple Present today; usually; during
Present Perfect since; for; yet; already; over the past few.......
Simple Past yesterday; in..; during;
Simple Continuous now; tomorrow;
Future tomorrow
Model test
In 1992, Bill Clinton has become president of the United States, beating his opponent by a wide
margin.
A B C D
What is wrong with this sentence? There is no agreement in meaning between the time marker and
the verb.

Explanation: in this sentence, the verb has become does not agree in meaning with the rest of the
sentence. The time marker in 1992 indicates that action in this sentence took place in the past only.
Has become indicates an action that began in the past and has continued to the present. Answer (A)
is the correct answer to the question because has become is incorrect.
Correction: became

9. PASSIVE AND ACTIVE SENTENCES

To help you to minimize the problems in this area, you need to be able to recognize the passive form
of the passive and to be able to determine when a passive verb rather than an active verb is needed in
a sentence.
The difference between an active and a passive verb is that the subject in an active sentence does
the action of the verb, and the subject in a passive sentence receives the action of the verb.

17
Two changes must be made to convert active to passive sentence:
1. The subject of the active sentence becomes the object of the passive sentence
2. The verb in the passive sentence is formed by putting the helping verb be in the same form in the
active sentence and then adding the past participle of this verb.

Look at this example:


Margaret wrote the letter
SUBJECT OBJECT

The letter was written by Margaret


SUBJECT OBJECT

It should be noted that in a passive sentence, by + object does not need to be included to have a
complete sentence. The following are both examples of correct sentences.
The letter was written yesterday by Margaret.
The letter was written yesterday.

Following are examples of passive errors that might be appear on the test.
1. The portrait is painting* by a famous artist.
(present participle painting is incorrect, it should be the past participle, painted)
2. The project will finished* by Tim
(some form of be is necessary for a passive sentence which has not been included yet, it
should be will be finished)

Model test
1. The assignment was done by most of the students.
A B C D
What is wrong with this sentence? The formation of the passive

Explanation: In this question, the passive is incorrectly formed. The past form of know has
been used when the past participle is needed. Answer (B) is the correct answer to the question
because was knew is incorrect.
Correction: was known

2. The people of this country have been expressed great concern about the environment.
A B C D
What is wrong with this sentence? The passive has been used when the active is needed
Explanation: In this question, the passive form of the verb is used. However, the sentence contains a
direct object, great concern. Direct objects come after active verbs, not passive verbs. Answer (C) is
the correct answer to this question because has been expressed is incorrect.
Correction: have expressed

3. The books and toys ____________ to the orphanage.


(A) being donated
(B) that were donated
(C) donated
(D) were donated

What is needed in this sentence? The passive


Explanation: In this question, the passive form of the verb is needed. Answer (A) does not contain a
finite verb. Answer (B) contains more than a finite verb. Answer (C) looks like an active finite verb.
However, piano cannot be the subject of donate. A piano cannot donate things. Only humans can
donate. Answer (D), which contains a correctly formed passive verb, indicates that someone donated
the piano. Using this passive verb form, the piano becomes the receiver, not the doer, of the action.
Answer (D) is the correct answer to this question.

18
10. MODALS AND MODAL-LIKE VERBS

In English sentences, modals (also called modal verbs, modal auxiliary verbs, modal auxiliaries) and
modal-like verbs are followed by the base form of the main verb. They give additional information
about the function of the main verb that follows them.
Model test
1. According to Joe, his puppy can recognizing its own name
A B C D
What is wrong with this sentence? The verb form that follows a modal
Explanation: In this sentence, the verb following can is in its present participle form. It should be in its
base form. Answer (B) is the correct answer to the question because can recognizing is incorrect.
Correction: can recognize
2. The book ________________ in the library.
(A) can be finding
(B) found
(C) to be found
(D) can be found

What is needed in this sentence? A modal passive


Explanation: In this question, a modal passive is needed. Answer (A) contains a present participle
where a past participle is needed. Answer (B) and (C) do not contain modal. Answer (D) is the correct
answer to the question because it contains the correct forms for this sentence.

11. SUBJECT – VERB AGREEMENT

Subject-Verb Agreement is actually simple: if the subject of a sentence is singular, the verb must be
singular; if the subject of a sentence is plural, the verb must be plural as well.
For example: The boy walks to school (singular/simple present tense)
The boys walk to school (plural/simple present tense)

However, you should be careful because the questions found in the ELPT test are often tricky. Verbs
don’t always follow the subject immediately. There are often phrases, inverted verbs, and other words
coming in between.

Strategy: Always identify the ‘real’ subject and verb

 Phrases

Prepositional phrase is a group of words beginning with a preposition (Ex.: to the main hall, on my
desk, during the session, etc). It usually contains a noun or verb word, so people often get confused in
identifying the subject and verb of the sentence. Be extra careful when the prepositional phrase is in
the middle of the sentence.

A. The climbers on the sheer face of the mountain needs to be rescued. (incorrect)
B. The climbers on the sheer face of the mountain need to be rescued. (correct)
S Prep. phrase V

In above example, the subject is the climbers, not the mountain. We can see that the word mountain
follows a preposition. It is a prepositional phrase. In the example, there are actually two prepositional
phrases: ‘on the sheer face’ and ‘of the mountain’. Either sheer face or the mountain is not the subject.

Other Examples:

Everybody are going to the theatre. (incorrect)


Everybody is going to the theatre. (correct)
S V
All of the book was interesting.
Singular
All of the books were interesting.
Plural
All of the information was interesting.
Uncount/singular

19
Model test
My friends and my teacher is coming to my show tonight.
A B C D

What is wrong with this sentence? Subject – Verb agreement


Explanation: In this question, the subject has two parts, my friends and teacher. These two together
make the plural subject. The form is is used for singular subject. Answer B is the correct answer to
the question because is coming is incorrect.
Correction: are coming

12. PREPOSITION
Many English sentences contain preposition. ELPT tests your understanding of preposition by:
1. Leaving out necessary prepositions or parts of compound prepositions;
2. Using incorrect prepositions, especially after certain verbs or adjectives;
3. Adding extra words to prepositional phrases.

Model test
1. Anne relaxes by she swimming in the lake every afternoon after class.
A B C D
What is wrong with this sentence? Extra parts have been added to a prepositional phrase.
Explanation: A prepositional phrase consists of a preposition and an object. By she swimming
contains a preposition, a subject, and an object. Answer (B) is the correct answer to the this question
because by she swimming is incorrect
Correction: by swimming
2. Last meeting, We talked ________________ the discovery of gold in California in the 1800s.
(A) on
(B) in
(C) for
(D) about

What is needed in this sentence? A preposition


Explanation: In this question, a verb + preposition combination is being tested. The verb talk is often
followed by the preposition about. Answer (D) is the correct answer to the question.
13. MAIN AND SUBORDINATE CLAUSE MARKERS

All English sentences contain at least one main clause. A main clause contains a subject and a verb and can
stand alone as a sentence

Example: Karina has a new book. = main clause


S V
Many English sentences also contain subordinate clauses. Like a main clause, a subordinate clause
contains a subject and a verb. However, a subordinate clause cannot stand alone as a sentence.
Example: Although Karina has a new book = subordinate clause
S V
There are three types of subordinate clauses in English:
1. Noun clauses
2. Adjective clauses
3. Adverb clauses

1. A noun clause functions as a subject, object, or complement in a sentence.


I like what you said.
Noun clause direct object

What you said is unbelievable


Noun clause direct object

2. An adjective clause functions as an adjective in a sentence.


I like the book that you gave me.
Adjective clause describing book

The man who gave you the book is my old friend.


Adjective clause describing the man

20
3. An adverb clause functions as an adverb in a sentence.
Tom is singing while Paul is dancing.
Adverb clause

Because it is raining outside, I am taking the umbrella with me.


Adverb clause

Clause markers (CM) are used to connect clauses in English. Each English clause type has its own special
clause markers.
Model test
1. Stephen is still coming to school even although it is raining.
A B C D
What is wrong with this sentence? The clause marker has extra part.
Explanation: A prepositional phrase consists of a preposition and an object. By she swimming
contains a preposition, a subject, and an object. Answer (B) is the correct answer to the this question
because by she swimming is incorrect
Correction: by swimming
2. Last meeting we talked ________________ the discovery of gold in California in the 1800s.
(A) on
(B) on
(C) for
(D) about

What is needed in this sentence? A preposition


Explanation: In this question, a verb + preposition combination is being tested. The verb talk is often
followed by the preposition about. Answer (D) is the correct answer to the question.

14. PREPOSITIONAL PHRASES AND SUBORDINATE CLAUSES

English prepositional phrase and subordinate clauses are easily confused. Confusion often happens
because, although they are formed differently, prepositional phrases and subordinate clauses can
function in similar ways in English.

Because of her financial difficulties, Anne could not go.


Prepositional phrase

Because she had financial difficulties, Anne could not go


Subordinate phrase

Model test
1. _____________ we have nothing in common, We can work well and submit the assignment on
time.
(A) Inspite of
(B) That
(C) Although
(D) Despite

What is needed in this sentence? An adverb clause marker


Explanation: In this question, there are two clauses, We have nothing in common and we can work
well and submit the assignment on time. A clause marker is needed at the beginning of the first
clause. Answers (A) and (D) do not contain a clause marker. Instead, they contain prepositions.
Answer (B) contains a clause marker. However, this clause marker would create a noun clause or an
adjective clause. The clause in this sentence is an adverb clause. Answer (C) is the correct answer
to this question because it contains a clause marker that can begin an adverb clause.
2. _____________ we have nothing in common, We can work well and submit the assignment on
time.
(A) Inspite of
(B) That
(C) Although
(D) Despite

21
What is needed in this sentence? An adverb clause marker
Explanation: In this question, there are two clauses, We have nothing in common and we can work
well and submit the assignment on time. A clause marker is needed at the beginning of the first
clause. Answers (A) and (D) do not contain a clause marker. Instead, they contain prepositions.
Answer (B) contains a clause marker. However, this clause marker would create a noun clause or an
adjective clause. The clause in this sentence is an adverb clause. Answer (C) is the correct answer
to this question because it contains a clause marker that can begin an adverb clause.

15. APPOSITIVE AND THE NOUN STRUCTURES THEY RENAME

An appositive is a noun that comes before or after another noun and has the same meaning.
Sally, the best student in the class, got an A on the exam.

In this example Sally is the subject of the sentence and the best student can easily be recognized as
an appositive phrase because of the noun student and because of the comma. Sally and the best
student are the same person.

Model test

1. ______, George, is attending the lecture.

(A) Right now


(B) Happily
(C) Because of the time
(D) My friend

What is needed in this sentence? A subject


Explanation: This sentence still needs a subject, because George is an appositive and not a Subject.
Answer (A), (B), and (C) is not a Subject. Therefore, the best answer is (D).

16. ONE-WORD –ING AND –ED ADJECTIVES


Verb forms ending in –ed and –ing can be used as adjectives. For example, the verbal adjectives
cleaned and cleaning come from the verb to clean.

For example

The woman cleans the car. (cleans is the verb of the sentence)
VERB

The cleaning woman worked on the car.


ADJ
(cleaning is a verbal adjective describing the woman)
The woman put the cleaned car back in the garage.
ADJ
(Cleaned is a verbal adjective describing car)
Remember:
The –ing adjective means that the noun it describes is DOING the action.
The –ed adjective means that the noun it describes is RECEIVING the action from the verb.
Look at the table below

Type Meaning Use Example


–ing active It does the action of the verb …the happily playing children.
(The children play)
–ed passive It receives the action of the verb …the frequently played record.
(Someone plays the record)

22
17. –ING AND –ED MODIFYING PHRASE
The –ing form of the verb (a present participle) can cause confusion because it can be either a part of
the verb or an adjective. It is part of the verb when it is preceded by some form of the verb be.
e.g. : The man is talking to his friend.

A present participle is an adjective when it is not accompanied by some form of the verb be.
e.g. : The man talking to his friend has a beard.

Model test

1. The child ______ playing in the yard is my son.


(A) now
(B) is
(C) he
(D) was

Explanation: ‘Playing in the yard’ is not part of the verb. It is an –ing modifying phrase. In this sentence
there is a complete subject (child) and a complete verb (is), so this sentence does not need another
subject or verb. The best answer is (A).

The –ed form of the verb (a past participle) can cause confusion because it can be either a part of the
verb or an adjective. It is part of the verb when it appears with have or be.
e.g.: The family has purchased a television.
The poem was written by Paul.

A past participle is an adjective when it is not accompanied by some form of the verb be or have.
e.g.: The television purchased yesterday was expensive.

Model test

2. The packages ______ mailed at the post office will arrive Monday.
(A) have
(B) were
(C) them
(D) just

Explanation: ‘Mailed at the post office’ is not part of the verb. It is an –ed modifying phrase. In this
sentence there is a complete subject (The packages) and a complete verb (will arrive), so this
sentence does not need another subject or verb. The best answer is (D).

18. GERUNDS AND INFINITIVES

Gerunds and infinitives are verb forms that can take the place of a noun in a sentence. The following
guidelines will help you figure out whether a gerund or infinitive is needed.

Following a verb (gerund or infinitive)


Both gerunds and infinitives can replace a noun as the object of a verb. Whether you use a
gerund or an infinitive depends on the main verb in the sentence.

I expect to have the report done by Friday. [INFINITIVE]


I anticipate having the report done by Friday. [GERUND]
Following a preposition (gerund only)
Gerunds can follow a preposition; infinitives cannot.

Can you touch your toes without bending your knees?


He was fined for driving over the speed limit.
She got the money by selling the car.

23
Note: Take care not to confuse the preposition "to" with an infinitive form, or with an auxiliary form
such as have to, used to, going to

He went back to writing his paper. [PREPOSITION + GERUND]


I used to live in Mexico. [AUXILIARY + VERB]
I want to go home. [VERB + INFINITIVE]

- Following an indirect object (infinitive only)


Some verbs are followed by a pronoun or noun referring to a person, and then an infinitive.
Gerunds cannot be used in this position.

I must ask you to reconsider your statement.


They begged her to stay for another term.
His findings caused him to investigate further.

Model test

In automotive companies, employees are rewarded for ______ with the firm for lengthy time periods.
(A) to stay
(B) staying
(C) to staying
(D) stay

What is needed in this sentence? A gerund


Explanation: The clue in this sentence is the preposition for. Preposition is followed by a gerund.
Answers (A), (C), and (D) are not prepositions. Therefore, Answer (B) is the best answer.

19. WORD FORM AND FUNCTION

The form of an English word must agree with its function in a sentence. For many English words
several forms are possible. Different word forms are created by endings, called suffixes, to the original
form of word.
Below are some of the common suffixes used to create different word forms.
NOUN (THING) SUFFIXES
-ism socialism -ment government
-ence excellence -ty specialty
-ion education -age marriage
-ness kindness -ship friendship
NOUN (PERSON) SUFFIXES
-er teacher -ist specialist
-or actor -ian mathematician
ADJECTIVE SUFFIXES
-ent excellent -ive effective
-ant important -ous dangerous
-ful beautiful -al natural
-ic panic -able capable
-less careless -ible sensible
VERB SUFFIXES
-ate educate -en darken
-ify beautify -ize organize
ADVERB SUFFIXES
-ly beautifully -ward backward

Model test

1. She told the truthful when we asked her about her past.
A B C D

What is wrong with this question? An adjective form has been used where a noun form is needed.

24
Explanation: In this question, a noun is needed after the article the. Truthful is not a noun; it is an
adjective. Answer (B) is the correct answer to this question because truthful is incorrect.
Correction: truth

2. The happily man spoke cheerfully about his family and friends.
A B C D

What is wrong with this sentence? An adverb form has been used where an adjective form is
needed.
Explanation: In this question, and adjective form is needed to describe man. Happily is an adverb
formed from the adjective happy. Answer (A) is the correct answer to this question because happily is
incorrect.
Correction: happy

20. EQUATIVE, COMPARATIVE AND SUPERLATIVE DEGREE

To make the equative, comparative, and superlative degree of adjectives and adverbs, the following
forms are used:
One syllable Two syllable Two or more syllable Adj. or Adv.
Adj. or Adv. Adj. or Adv.
ending in -y
Equative as … as as … as as … as
as tall as as happy as as beautiful as
Comparative -er than … -er than more/less … than
taller than happier than more beautiful than
Superlative the … -est the … -est the most/least …
the tallest the happiest the most beautiful

NOTE: Some words have irregular degree forms. The most common of these are:

ADJECTIVE ADVERBS
good better best well better best
bad worse worst badly worse worst
much/many more most much more most
little less least little less least
far farther farthest far farther farthest
far further furthest far further furthest

 The equative degree is used to show equality


Annie is as tall as Jacob.
This sentence tells us that Annie and Jacob are the same height.

 The comparative degree is used to compare two things that are not equal.
Annie is taller than Jacob
This sentence tells us that Annie’s height are not the same. In addition, it tells us that Annie’s
height is greater than Jacob’s.

 The superlative degree is used to compare three or more things that are not equal.
Annie is the tallest student in the class.
This sentence tells us that there are more than two students in the class and that Annie’s
height is the greatest of all the students.

Model test

1. Although Tom’s paper was longer than Alice’s, Alice’s paper was more insightful that Tom’s.
A B C D
What is wrong in this sentence? The comparative degree has been incorrectly formed.

25
Explanation: In this question, there are two comparative structures. The first, longer than, is correctly
formed. However, the second, more insightful that, is incorrectly formed. That is not used in forming
the comparative degree. Answer (D) is the correct answer to this question because more insightful that
is incorrect.
Correction: more insightful than

2. Professor Clark’s chemistry class is __________ Professor Smith’s.


A. more than difficult
B. to the difficult
C. as difficult as
D. the most difficult

What is needed in this sentence? the equative degree


Explanation: In this question, two classes are being compared. Therefore, either the comparative or
the equative degree is needed. Answer (A) contains an incorrectly formed comparative degree.
Answer (B) contains a prepositional phrase. Answer (D) contains the superlative degree. Answer (C) is
the correct answer to this question because it contains a correctly formed equative degree.

21. STANDARD WORD ORDER

The standard word order of the principal parts of English sentence is:

SUBJECT + VERB (+OBJECT AND/OR + COMPLEMENT)

The subject comes before the verb. The verb comes before its object and/or before the complement of
the sentence.

Kathy is eating
S V
Kathy is happy
S V Comp
Kathy is eating chocolate
S V DO
Model test

1. Karen’s home __________ on a busy downtown street


(A) Small, is an apartment
(B) A small apartment is
(C) An apartment is small
(D) Is a small apartment

What is needed in this sentence? A verb and a complement

Explanation: In this question, the verb is missing. The verb BE is contained in all of the answer
choices. A complement, a small apartment is also contained in all of the sentences. However, the
word order is incorrect in all of the answer choices except for answer (D). Answer (D) is the correct
answer to this question.

22. WORD ORDER IN SUBORDINATE CLAUSES BEGINNING WITH


QUESTIONS WORDS

Question words such as who, what, when (ever), where (ever), why, how (ever), how long, how much,
and how many occur in many English sentences. These words are often used to begin subordinate
clauses that do not ask a direct question. The sentences that contain these clauses are not followed
by a question mark (?). In these clauses the subject and the verb are not inverted.

I don’t know who she is


Noun Clause
When she arrives, please show her in.
Adverb clause
Tell me the reason why she didn’t come

26
Adjective clause
Model test

1. I don’t know why ________________.


(A) did she leave me
(B) she leaves me
(C) she did leave me
(D) she left me

What is needed in this sentence? A subject and a verb

Explanation: In this question, the word why is used to connect the two clauses so a subject and verb
are needed after this connector; this is not a question, so the subject and verb should not be inverted.
The best answer is therefore answer (D).

23. INVERTED SUBJECT-VERB WORD ORDER


WITH SPECIAL EXPRESSIONS AND IN CONDITIONAL SENTENCE
The standard word order of subject + verb (+ object and/or + complement) is not followed in all English
Sentences. In certain situations, inverted subject-verb word order is used. That is, the subject of a
sentence is placed after the first helping verb or other BE. If there is no verb BE or if there is no
helping verb, the helping verb DO is added as the first verb of the sentence. This inverted subject-verb
word order is most common in direct questions, but it is also common in other situations.

1. After special expressions of location:


On the beach were five beach umbrellas
Expression of location V S
Nowhere did he see her
Expression of location first V S
2. After special negative (no, not, and never) and almost negative (hardly, rarely, scarcely, not
only, no time, barely, only, seldom, etc) expressions:
Never had I seen such a glorious sight
Negative first V S
Only after he saw her did he understand *
Almost negative first V S

*NOTE: When the negative or almost negative expression is a part of a subordinate clause,
the subject and verb of subordinate clause are not inverted. The subject and the verb of the
main clause are inverted.

3. After the special expressions so and either:


So happy was she that she danced around the room
V S
I liked the coffee, so did Mike
V S
I didn’t like the coffee, and neither did Mike
V S

4. In conditional sentences that do not begin with if:


With if: If he had seen you, he would have greeted you
S V
Without if: Had he seen you, he would have greeted you first
V S

Model test

1. Rarely____________ happy.
(A) John is ever
(B) Is john ever
(C) Ever John is
(D) John ever is

What is needed in this sentence? An inverted subject and verb

27
Explanation: In this question, rarely is the first word of the sentence. Inverted subject-verb word order
occurs after rarely. Answer (B) is the correct answer to this question. It is the only answer choice that
contains inverted subject-verb word order.

2. ____________, he would have understood the movie better.


(A) Carl had read the book
(B) The book had been read by Carl
(C) Had Carl read the book
(D) Read the book had Carl

What is needed in this sentence? A conditional that does not begin with if

Explanation: In this question, each answer choice contains parts of a conditional without if. In these
types of conditionals, inverted subject-verb word order is needed. Answer (A) does not contain
inverted word order. Answer (B) contains a passive construction. This is not the inverted structure that
is needed. Answer (D) contains inverted word order, but using the wrong part of the verb. Answer (C)
is the correct answer to the question because it contains correct inverted subject-verb word order.

24. PARALLEL STRUCTURE in Comparisons and in Series


Joined By And, But, or Or

When words, phrases, or clauses have parallel (similar) functions in English sentences, they should
have parallel structure as well. It is important to maintain parallel structure in English sentences.

Clarice is beautiful, smart, and talented.


Adj Adj Adj

He won the lottery, quit his job, and bought presents for all his friends.
Verb phrase Verb phrase Verb phrase
You should be especially careful about parallel structure when:

1. Parallel structure in words, phrases, and clauses used in series joined by and, but, or or:

Incorrect: He likes to swim, to hike, and riding his bike.


infinitive infinitive gerund
Correct: He likes to swim, to hike, and to ride his bike.
infinitive infinitive infinitive

Incorrect: She went home because she needed money and due to her mother’s illness.
Subordinate clause Prepositional phrase
Correct: She went home because she needed money and because her mother was ill.
Subordinate clause Subordinate clause

Incorrect: Her cheerful outward manner and she told interesting stories made her very popular.
Noun phrase Main clause
Correct: Her cheerful outward manner and her interesting stories made her very popular.
Noun phrase Noun phrase

2. Parallel structure with equative (as…as) and comparative (-er than/more…than) constructions
and with other expressions of comparison (e.g., similar to, the same as):

Incorrect: How to pronounce English is more difficult than reading it.


Infinitive phrase Gerund phrase
Correct: Pronouncing English is more difficult than reading it.
Gerund phrase Gerund phrase

In comparisons, you must be especially careful that the two compared things are parallel in both
meaning and structure.

Incorrect: The shelves in Durick library are taller than Norwich Library
In this sentence, shelves and Norwich Library are being compared. These two nouns are
not comparable (parallel in meaning) even though they are parallel in structure.

28
Correct: The shelves in Durick library are taller than those in Norwich Library
In this sentence, shelves are being compared to shelves, referred to by the pronoun
those. These two structures are parallel in meaning and in structure.
Model test

1. Lance’s efforts led to a promotion,___________, and an award of $10,000.


(A) His peers recognized him
(B) recognition by his peers
(C) he received recognitions from his peers
(D) to receive recognition from his peers

What is needed in this sentence? A noun phrase parallel to a promotion and an award

Explanation: In this sentence, noun phrase are being joined in a series with and. Answers (A) and (C)
contain more than a noun phrase. They contain complete sentences. Answer (D) contains an infinitive
phrase. Answer (B) is the correct answer to this question. It contains a noun phrase that is paralleled
in structure to a promotion and an award.

2. Sleeping well is as important to good health as to eat well


A B C D

What is wrong with this sentence? A comparison is being made between two structures which are
not parallel

Explanation: In this question, the gerund sleeping is compared to the infinitive to eat. These two
structures are not parallel. Answer (D) is the correct answer to this question because to eat is
incorrect.
Correction: eating

25. PAIRED EXPRESSION

There are pairs of expressions in English which can be especially confusing. These paired
expressions are:
both….and
either….or
neither….nor
not only….but also

Paired expressions can act as clause markers, or they can function to join words or phrases.

As clause markers:
Either he will go to the movie, or he will go to the play
CM Clause CM Clause

Joining phrases:
He will go either to the movie or to the play
C Prep phrase CM Prep phrase

Joining words:
He will go either today or tomorrow
CM Word CM Word

Paired expressions need to be checked for a variety of things. Parallelism is important for paired
expressions. The same grammatical from should follow each word of a paired expressions.

Incorrect: Laura both enjoys books and music


Verb Noun

Correct: Laura enjoys both books and music


Noun Noun

29
Model Test

1. We must ___________ but also our clothes.


(A) Not only pack our books
(B) Pack not only our books
(C) Our books not only pack
(D) Not our only books pack

What is needed in this sentence? The first half of a paired expression + a noun phrase

Explanation: In this question, the second half of a paired expression, but also, is present. But also is
followed by the noun phrase our clothes. To keep parallel structure in this sentence, not only followed
by a noun phrase is needed. Answers (A) and (C) contain not only followed by a verb. Answer (D)
separates not from only, which creates an incorrect form. Answer (B) is the correct answer to the
question because it contains not only followed by a noun phrase.

2. Both Leo also Margaret were waiting for the bus when the storm began
A B C D

What is wrong with this sentence? A paired expression has been incorrectly formed

Explanation: In this question, the first part of a paired expression, both, is used. However, the second
part of this expression is missing and has been replaced by also. Answer (A) is the correct answer to
the question because also is incorrect.
Correction: and

26. CONFUSING WORDS AND EXPRESSION

There are words and expressions in English which are especially confusing, because they sound very
much alike and/or because they have very similar functions in English sentences.
Incorrect: He was formally a teacher, but now he is a car dealer
Correct: He was formerly a teacher, but now he is a car dealer
Formally and formerly in this pair of sentences sound very much alike and have similar function.
However, their meanings are quite different. Formally means officially and does not have the meaning
necessary for this sentence. Formerly, which means before, has the correct meaning for this sentence.
Incorrect: Melissa often does a cake for dessert
Correct: Melissa often makes a cake for dessert
Does and make in this pair of sentences do not sound alike. However, they both function as verb, and
they have some similarities in meaning. The verb DO, however, often expresses the idea of
performing of completing. Make often expresses the idea of creating or constructing. The difference in
meaning between make and do is great enough that they are not interchangeable. Since Mellissa had
created rather than performed the cake, make is the correct verb for this sentence.

Model Test

1. The tests in this class are less difficult than tests in another classes
A B C D

What is wrong with this sentence? A word that sounds like the correct word has been substituted
for the correct word
Explanation: In this question, another has been used to describe classes. However, another is used
only with singular, countable, indefinite nouns. Answer (D) is the correct answer to this question
because another is incorrect.
Correction: other

30
READING COMPREHENSION
1. SKIMMING

Skimming means reading quickly for general meaning. You skim reading material to find out about the
topic, the main ideas, and the general organization of a passage.

2. SCANNING

Scanning means knowing what information you need to find before you read. Then your eyes move
quickly to find that particular information. You can scan when you are looking for a fact or a detail or a
particular vocabulary word.

Model Test

People are still reading books, and lots of them. The number of new books published keeps
growing-90,000 new titles in the United States during the last year by the latest industry count.
The number is double the total output two decades ago. Reading is still an important part of
our lives.

1. What is this passage about?


A. The amount of reading people do
B. Book reviews
C. Leisure time activities
D. A popular new book

2. How many new books were published in the United States last year?
A. 60,000
B. 900
C. 9,000
D. 90,000

Explanation
The correct answer to question 1 is (A) The amount of reading people do. By reading the first
sentence of the passage you find the phrases reading books and lots of them. As you quickly read the
rest of the paragraph you find these words and ideas repeated in different ways. You could answer
the question by quickly skimming the passage for the general idea.
The correct answer to question 2 is (D) 90,000. The question asks how many, and this tells you to
look for a number. By scanning the passage for a number, you are able to find the correct answer of
90,000.

3. TOPIC AND MAIN IDEAS

Almost all passages contain one question about either the topic or subject, or the main ideas of a
reading passage. This type of question is usually the first question about the passage. These
questions ask about the whole passage, not just a part of the passage. In order to correctly answer
these questions, you should use the skimming and scanning strategies.

To answer questions about the topic and the main ideas of a passage, follow these steps:
1. Read the first several sentences for the topic and main ideas.
2. Read the last sentences for the conclusion and a possible restatement of the topic and the
main ideas.
3. Skim the rest of the passage for the key words that will confirm the topic and the main ideas
and show the organization of the passage.
4. Read the first question about the passage and answer it.
5. Read the answer choices. Eliminate any answers that are definitely wrong, and choose the
best answer from the remaining ones.

31
Model Test

In the critical area of food production, new cooperative efforts in agricultural


research and development are paying off. Food scientists are discovering that humankind is
nowhere near the limits of plant, livestock, and soil productivity. There is still room to boost
yields and learn how to use more efficiently the earth’s acreage for animal and crop
husbandry. Investigation of irrigation procedures, pest control, intercropping, and multi
cropping are several areas that are providing useful information for the world’s farmers.
1. What is the topic of the passage?
A. Agricultural research and food production.
B. Farmers throughout the world.
C. Food scientists.
D. Investigation of irrigation procedures.

2. The passage supports which of the following statements?


A. New information from agricultural research can increase world food production.
B. Food production is declining around the world.
C. Agricultural research and development will be expensive.
D. There are few initiatives to help today’s farmers.
Explanation
Question 1 asks about the topic of the passage, which is the most general statement you make about
the passage. In the first sentence of this passage the phrases food production and agricultural
research and development give you this basic concept. Answer (B), (C) and (D) are too specific to be
the topic of the passage. Answer (A) is the most general statement in the passage
Question 2 asks you about idea of the passage. A main idea of a passage tells you something more
about the topic. In this passage the author tells us that efforts in agricultural research and
development (in the area of food production) are paying off, which means they are beneficial. The
correct answer is (A) New information from agricultural research can increase world food production.
Answer (B) and (D) are not true, and answer (C) is not mentioned in the passage.

4. VOCABULARY IN CONTEXT

The Reading Section tests your knowledge of vocabulary by asking questions about certain words in
the reading passages. For the most part, you can answer these questions about word meanings by
using the context of the sentence in which the word was found or of the passage in general.

Model Test

The hippopotamus spends as much time in the water as on land. It swims and dives well,
but prefers to spend its days submerged in the water with only its eyes and ears sticking out. It
lives in the swamps, streams and marshes of tropical Africa.
1. As used in lines 2 and 3, the phrase “submerged in the water” means that the animal is
A. swimming in the water
B. standing covered with water
C. floating in the water
D. wading in water

Explanation
Since all of the answers choices involve the water, you must read carefully for context clues in the
sentence. The sentence states that it spends its days .... in the water with only its eyes and ears
sticking out. Using the full context of the sentence, we read that the hippo swims and dives well, but
prefers to spend its days submerged ....; therefore answer (A) swimming .... is not the same as
submerged. You know that answers (C) floating on the water and (D) wading in water both expose
more than the eyes and ears of the hippo, and are incorrect. The correct answer choice is (B)
standing covered with water.
You reach this conclusion by inferring information from the passage and from the answer choices and
by using logical reasoning.
NOTE: In some questions, however, you cannot rely on context for figuring out the meaning of the
words. In this case your knowledge of synonyms, word forms, Latin and Greek roots, prefixes, and
suffixes will help you to answer the questions about word meanings.

32
In the following example sentences, common context cues in written discourse are illustrated for the
word “adversity”.

Contrast It was adversity, not prosperity, that shaped the lives of early American pioneers.
Example Pioneer women met adversity head on; they used paper for windows, carried water for
miles for the weekly washing, and made do with corn meal rather than flour for the
daily baking.
Definition The adversities for the winter – that is, the hardships and calamities that the family
lived with daily – disappeared with the warmth of spring.
Appositive Locusts (the dreaded migratory grasshoppers of the prairies) could destroy a family’s
whole corn crop in a matter of minutes. From these adversities, the trials and
tribulations of the times, came the legends of the hardy American pioneers.
Series The diaries of the early farmers told of hardships, calamities, and adversities that
made their lives miserable: Lack of water for daily use, sudden epidemics that wiped
out whole families, and severe blizzards that kept people house-bound for weeks.
Parallelism The settlers never gave up. They struggled through the early years, dealt with their
difficulties, and in the end, overcame their adversities.
WH marker (where, when, which, who, whom, that) Relative pronouns often suggest the meaning
of words and phrases.
e.g.,The cold-cellar was where the farmers stored their fruits and vegetables for the
winter.
Reason logically to make sense of unfamiliar words and phrases in a reading passage by using
other words in the sentence and the passage.
e.g., The settlers never gave up. They struggled through the early years, dealt with
their difficulties, and in the end, overcame their adversities.
The general meaning of adversities can be deduced by considering the “core”
meanings that the other key words in the context share: persistence (never
gave up, struggled, overcame) and hardship (struggle, difficulties).
Use your knowledge of the world and personal experience.
e.g., Pioneer women met adversity head on; they used paper for windows, carried
water for miles for the weekly washing, and made do with corn meal rather than
flour for the daily baking.
What you know about windows, washing, and baking from your personal
knowledge and experience will help you to infer that the life of the pioneer
woman was very difficult and that adversity must include this concept of
difficulty or hardship.

5. PURPOSE AND ORGANIZATIONAL PATTERN

Some Reading Comprehension questions ask about the author’s purpose for the passage or about the
organization of the passage.

The purpose of passage is the reason the author wrote the passage or the intent of the author in
writing the passage. The organizational pattern of a reading passage is the way that the author
arranges the information to carry out his or her writing rather than the purpose of the whole passage.

Model Test

The art of writing itself is a good example of what students of the past call independent
invention, since systems of writing have evolved in isolation at different times in different parts
of the world. For example, one system—the Chinese ideogram—can be traced to its origin in
archaic signs engraved on the scapular bones of sheep or the shells of turtles in the second
millennium B.C. as a means of asking questions of heaven. Roughly 1,000 years later an
entirely independent system of writing arose halfway around the world in Mesoamerica. It
combined a simple system of numerical notation with complex hieroglyphs and was
principally used to indicate the dates of various events according to an elaborate calendrical
system.

1. What is the purpose of this passage?


A. To show that writing is an example of independent invention.
B. To explain the origin of writing
C. To describe two systems of writing
D. To compare writing in China to writing in Mesoamerica

33
2. Which of the following best describes the organization of the passage?
A. A comparison of two competing systems.
B. An examination of a problem
C. A statement supported by examples.
D. A chronological development.

3. Where in the passage does the author describe the early writing system of Mesoamerica?
A. Lines 1-3
B. Lines 3-6
C. Lines 5-7
D. Lines 7-9

Explanation
The correct answer to Question 1 is (A). The sentence …writing is an example of independent
invention contains both topic and main idea of the passage. The use of examples to support this is
represented by To show that. Answer (B) and (C) are not true, and answer (D) is not specific enough
to this passage.

The correct answer to Question 2 is (C). The first sentence of the passage states that systems of
writing have evolved in isolation at different times in different parts of the world. From this we
understand the topic (systems of writing) and the main idea (evolved in isolation). Answer (C) correctly
represents the organization of the passage answer (A) is incorrect because there is no comparison
made, and the systems of writing are not competing. Answer (B) is incorrect because the main idea is
not a problem but a statement of fact. Answer (D) is incorrect because although dates and years are
mentioned in the passage, they are used to support the main idea and not the organizational pattern.
The correct answer to Question 3 is (D). in line 7 we find the word Mesoamerica. The question asks
about the description of the writing system of Mesoamerica, which is found in line 7-9.

6. REFERENCE WORDS

Reference words are those words in a passage that refer back to words or phrases mentioned earlier
in the passage or refer forward to words or phrases that will be introduced.

In determining the referent (the word or phrase that reference words refer to), you should use both the
structure and the meaning of the sentence.

Throughout the year, chimpanzee food is quite varied, but it is mainly vegetable material. At
times, however, the Gombe Park is loaded with insects-termites, ants, caterpillars- and the
chimpanzees will eat huge numbers of them. The chimpanzees’ really remarkable behavior
appears when they gather termites. According to Suzuki and van Lawick-Goodall, when
chimpanzees see that termites have pushed open their tunnels on the surface, they will go
off to find a suitable termiting tool. It may look simple, but the job takes skill and patience.

1. The word “their” in line 6 refers to?


A. the termites’
B. the tunnels’
C. the chimpanzees’
D. Suzuki’s and van Lawick-Goodall’s

2. The pronoun ‘they” in line 6 refers to?


A. Suzuki and van Lawick-Goodall
B. termites
C. tools
D. chimpanzees

3. The pronoun “It” in line 6 refers to


A. suitable termite tool
B. The job
C. Skill

34
D. patience

Explanation
The answer to Question 1 is (A) the termites’. This question might be confusing because you know
that their can refer to a plural noun. The sentence, Suzuki and ........chimpanzees, .... termites are all
plural concepts. Logical reasoning will tell you t hat termites are the ones that live in tunnels.

The answer to Question2 is (D) Chimpanzees. In this case, the plural pronoun they could refer to
either chimpanzees or termites. The meaning of the whole passage, however, makes it clear that the
chimpanzees are looking for a tool to gather termites, and they refers to chimpanzees.

The answer to Question 3 is (B) the job. The pronoun it could be possibly refer to any singular object,
and at first glance you might choose a suitable tool. However, as you read on, it becomes clear that
what looks simple is the job. In this sentence the pronoun refers forward.

7. DETAIL and FACTUAL INFORMATION


There are two types of fact and detail questions: questions about what IS true according to the
information and questions about what IS NOT true. Answer choices for questions about facts and
details may use the exact words of the passage, but more often they are restatements of the
information and require that you know synonyms and related words.

Model Test

Throughout the year, chimpanzee food is quite varied, but it is mainly vegetable material. At
times, however, the Gombe Park is loaded with insects-termites, ants, caterpillars- and the
chimpanzees will eat huge numbers of them. The chimpanzees’ really remarkable behavior
appears when they gather termites. According to Suzuki and van Lawick-Goodall, when
chimpanzees see that termites have pushed open their tunnels on the surface, they will go off
to find a suitable termiting tool. It may look simple, but the job takes skill and patience.

1. According to the passage, which of the following are NOT mentioned as part of the
chimpanzees’ diet?
A. Termites
B. Vegetable materials
C. Ants
D. Mosquitoes

2. The author states in the passage that the chimpanzees’s most remarkable behavior can
best be seen
A. when they are hungry
B. as they are resting
C. when they are looking for termites
D. in the spring

3. According to the author, when chimpanzees gather termites they show


A. a dependence on each other
B. remarkable strength
C. understanding and caring
D. ability and persistence

Explanation
The correct answer to Question 1 is (D) Mosquitoes. This question asks to you to identify the answer
that is NOT in the passage. By knowing where in the passage the food chimpanzees eat is mentioned,
you can quickly look at those sentences and match the items in the sentence with those in the answer
choices. Mosquitoes are not mentioned in the passage.

NOTE: in the questions that ask what is NOT in the passage, information that is true is not the correct
answer.
The correct answer to Question 2 is (C) when they are looking for termites. To answer this question
you need to match the words chimpanzees’ most remarkable behavior in the question with those
words in the passage. This will tell you in what part of the passage you will find the answer. After
careful reading of the sentence, you can match the information in the passage with the answer choice.

35
In this case the passage states when they gather termites, and restatement of this is found in answer
choice (C).
The correct answer to Question 3 is (D) ability and persistence. In this question you are asked to find
what chimpanzees show rather than do when they gather termites. The last sentence of the passage
states that the job (of gathering termites) takes skill and patience. Scanning the answer choices will tell
you that the best restatement of the information in the passage is ability and persistence.

8. MAKING INFERENCE

Making inference (a prediction or conclusion) ask you to use your understanding of the facts and
details which are directly stated in a reading passage. Information that is not directly stated in the
passage is said to be implied by the author. Questions about implied information may be about a part
of the passage or about what came before or will come after the passage. You may be asked to draw
conclusions about the passage itself, or to make predictions about another related situation.

Model Test

Think about the following sentence and questions:

In last year’s competition of the five contestants chosen, one was from White Springs, two
from other towns in Idaho, and the rest from neighboring areas of the Pacific Northwest.

Using the facts of the text itself, you can make several inferences (conclusions based on facts)
about this sentence.

1. What or where is White Springs?


We can infer White Springs is a town in Idaho based on the phrase other towns in Idaho.
2. Where is Idaho?
We can infer that both White Springs in Idaho are in the Pacific Northwest, based on from
neighboring areas of the Pacific Northwest.
3. How many contestants are from the rest of the Pacific Northwest?
We can infer there where two contestants from the neighboring areas of the Pacific North
West by using simple arithmetic.

36
Practice Test Listening
Directions: In Part A you will hear short conversations between two people. After each conversation,
you will hear a question about the conversation. The conversations and questions will not be repeated.
After you hear a question, read the four possible answers in your test book and choose the best
answer. Then, on your answer sheet, find the number of the question and fill in the space that
corresponds to the letter of the answer you have chosen.

1. A. The coffee is much better this


morning.
B. The coffee tastes extremely good. 8. A. Housing within his budget is hard to
C. The coffee isn’t very good. locate.
D. This morning he definitely wants some B. It’s hard to find his house in New
coffee. York.
C. He can’t afford to move his house to
2. A. The classes meet in an hour and a New York.
half. D. Housing in New York is unavailable.
B. The class meets three hours per
week. 9. A. The boss was working on the
C. Each half of the class is an hour reports.
long. B. He would have to finish the reports
D. Two times a week the class meets for before the end of next month.
an hour. C. He was directed to stay late and
finish some work.
3. A. A few minutes ago, the flight D. He could finish the reports at home.
departed.
B. The fight will start in a while. 10. A. The boisterous students made
C. They are frightened about the the teacher.
departure. B. The teacher angered the students
D. The plane is going to take off soon. with the exam results.
C. The students were angry that the
4. A. He hasn’t yet begun his project. teacher was around.
B. He’s supposed to do his science D. The angered students complained
project next week. to the teacher.
C. He needs to start working on
changing the due date. 11. A. The prices are reasonable.
D. He’s been working steadily on his B. The store is too far out of town.
science project. C. He would like the woman to repeat
what she said.
5. A. At the post office D. He agrees with the woman.
B. In a florist shop
C. In a restaurant 12. A. It’s rained unusually hard this year.
D. In a hospital delivery room B. There hasn’t been any rain for many
years.
6. A. The professor drowned the cell in a C. It’s been many years since it rained.
lab. D. He doesn’t like rain.
B. The lecture was long and boring.
C. The professor divided the lecture 13. A. He needs to do a better job writing
into parts. questions.
D. The biologist tried to sell the results B. He certainly must make his writing
of the experiment. better.
C. Without the questions, he cannot
7. A. She needs to get a driver’s license. write the answers.
B. Two pieces of identification are D. He needs to understand the written
necessary. questions better.
C. The man should check to see if he
needs credit. 14. A. The agent was standing in line with
D. A credit card can be used to get a his passport.
driver’s license. B. The line to get new passports is very
long.

37
C. The woman must wait her turn to
get her passport checked.
D. He can check her passport instead
of the agent.
15. A. He couldn’t finish closing the library 22. A. The repairs that the mechanic had
book. indicated were already made.
B. He hadn’t finished the library B. The car is going to need a lot of
assignment, but he was close. repairs.
C. He was working on the assignment C. Buying a new car would be quite
when the library closed. expensive.
D. His homework was incomplete D. The mechanic extended the repair
because the library wasn’t open. warranty.

16. A. All the lawyer’s preparation did no 23. A. Betty wrote the letter as directed.
good. B. The directions were given to Betty
B. The lawyer prepared nothing for in a letter.
the case. C. Betty will follow the instructions later.
C. It wasn’t work for the lawyer to D. Betty worked exactly as instructed.
prepare for the case.
D. The lawyer didn’t work to prepare 24. A. Walter had a lack of success with his
for the case. business.
B. Walter failed in business.
C. Walter’s new company is doing
17. A. He thinks the papers should be rather well.
turned in next week. D. Walter hoped to succeed in
B. The history class begins next week. business.
C. He has already done the paper for
next week. 25. A. He should put the organ in the
D. The papers are not due next week. closet.
B. The closet has already been
18. A. He’s not really happy. organized.
B. The contractor’s work was C. He needs to rearrange the closet.
satisfactory. D. He wishes the closet were closer.
C. He would rather work with the
contractor himself. 26. A. She didn’t do the work.
D. He was already contacted about B. She gave the assignment her best
the work. effort.
C. She finished the assignment even
19. A. The man should try another type of though it was difficult.
paper. D. She gave the man a signal.
B. The man should locate a typist 27. A. She said some terrible things.
tomorrow morning. B. She didn’t say anything nice.
C. The man should make a tape in the C. She didn’t have any nice things.
morning. D. She said really wonderful things.
D. The man should complete the paper
without help.
28. A. New employees are rarely initiated
20. A. She’d like some pie. into the company.
B. It’s easy to buy it. B. New workers don’t generally
C. The task the man’s working on isn’t undertake actions on their own.
difficult. C. New employees are initially rated.
D. It’s easier to prepare pie than do D. It’s rare for employees to make new
what the man is doing. suggestions.

21. A. He reported that the time for the 29. A. The woman is more than a week
budget meeting had been set. late.
B. He is always late in submitting his B. The children would have wrecked
accounting figures. the house later.
C. He never manages to budget his C. The woman was so late that she was
time well. a wreck.
D. He is never too late in turning in his D. He’s glad that she was not any later.
reports.
30. A. He had not gone to the store.

38
B. He was still at the market. shopping.
C. He was going to take care of the D. He always went to the market.
Part B

Direction: In this part of the test, you will hear longer conversations. After each conversation, you will
hear several questions. The conversations and questions will not be repeated.

After you hear a question, read the four possible answers in your test book and choose the best
answer. Then, on your answer sheet, find the number of the question and fill in the space that
corresponds to the letter of the answer you have chosen.

Remember, you are not allowed to take notes or write in your test book.

31. A. She’s a senior.


B. She’s junior. 35. A. In a book
C. She’s transfer student. B. From a television program
D. She’s graduate student. C. During a trip that she took
D. From a lecture
32. A. How to transfer to a junior college
B. How to find his way around campus 36. A. To communicate with other dolphins
C. What courses are required for a B. To recognize objects in the water
literature major C. To learn human language
D. Who won the campus election D. To express fear

33. A. Three 37. A. Five


B. Five B. Fifteen
C. Eight C. Fifty
D. Ten D. Five hundred

34. A. American literature 38. A. It is limited.


B. World literature B. It is greater than human intelligence.
C. Literary analysis C. It is less than previously thought.
D. Surveying D. We are beginning to learn how
much they have.

39
Part C

Directions: In this of the test, you will hear several talks. After each talk, you will hear some questions.
The talks and questions will not be repeated.

After you hear a question, read the four possible answer in your test book and choose the best
answer. Then, on your answer sheet, find the number of the question and fill in the space that
corresponds to the letter of the answer you have chosen.

Here is an example.

On the recording, you hear:


(narrator) Listen to an instructor talk to his class about painting.
(man) Artist Grant Wood was a guiding force in the school of painting known as American regionalist,
a style reflecting the distinctive characteristics of art from rural areas of the United States. Wood
began drawing animals on the family farm at the age of three, and when he was thirty-eight one of his
paintings received a remarkable amount of public notice and acclaim. This painting, called “American
Gothic,” is a starkly simple depiction of a serious couple staring directly out at the viewer.

Now listen to a sample question.

(narrator) What style of painting is known as American regionalist?

In Your test book, you read: (A) Art from America’s inner cities
(B) Art from the central region of the United States
(C) Art from various urban areas in the United States
(D) Art from rural sections of America

The best answer to the question “What style of painting is known as American regionalist?” is (D), “Art
from rural sections of America.” Therefore, the correct choice is (D).

Now Listen to another sample question.


(narrator) What is the name of Wood’s most successful painting?

In your test book, you read: (A) “American Regionalist”


(B) “The family Farm in Iowa”
(C) “American Gothic”
(D) “A Serious Couple”

The best answer to the question, “What is the name of Wood’s most successful painting?” is (C),
“American Gothic.” Therefore, the correct choice is (C).
Remember, you are not allowed to take notes or write in your test book.

39. A. To protect its members C. Only in California


B. To save the natural environment D. Only in the Sierra Nevadas
C. To honor the memory of John Muir
D. To improve San Francisco’s natural 43. A. Students signing up for athletic
beauty teams
B. Students going on a tour of a
40. A. For less than a year university campus
B. Only for a decade C. Students playing various sports
C. For more than a century D. Students attending a university
D. For at least two centuries dedication ceremony

41. A. San Francisco 44. A. Membership on an athletic team


B. All fifty states B. Enrollment in an exercise class
C. The Sierra Nevadas C. A valid student identification card
D. The eastern United States D. Permission from a faculty member

42. A. All over the world 45. A. To the tennis courts


B. In the entire United States B. To the arena

40
C. To the gymnasium
D. To the Athletic Department office

46. A. Go to the Art Center


B. Sign up for sports classes
C. Visit the exercise room
D. Watch a football game

47. A. Science
B. Art
C. Literature
D. Music

48. A. They are completely different.


B. They are somewhat similar but have
an essential difference.
C. They are exactly the same in all
respects.
D. They are unrelated.

49. A. Objective
B. Idealistic
C. Philosophical
D. Environmental

50. A. Heredity
B. Environment
C. Idealism
D. Natural laws

41
Practice Test SWE
1. Kansas_____ at the geographical center of the United States.
(A) it is located
(B) the location is
(C) its location
(D) is located

2. ______first pizza restaurant opened in New York City in 1895.


(A) The
(B) It was the
(C) At the
(D) It was at the

3. ______ without a backbone is the Atlantic squid.


(A) For the largest creature
(B) It is the largest creature
(C) The largest creature
(D) The largest creature is

4. The last star on the handle of_____ Polaris, or the orth Star.
(A) to call the Little Dipper
(B) calling the Little Dipper
(C) the Little Dipper calling
(D) the Little Dipper is called

5. Lis Moses, ______________, grows beautiful vegetables in her garden


(A) is my next-door neighbour.
(B) of my next-door neighbour.
(C) whom my next door neighbour.
(D) my next-door neighbour.

6. Eugene Debs ran for the presidency of the United States five times, _____was never elected.
(A) he
(B) but he
(C) for him
(D) for his

7. The oldest known daggers are ones that Neolithic humans______out of flint
(A) skillfully chipped
(B) skillful chips
(C) chipping skillfully
(D) chips are skillful

8. Woodrow Wilson served as president of Princeton_______was elected precident of the United


States.
(A) before
(B) to him
(C) he
(D) before he

9. Sound waves are produced when____


(A) objects of vibrate
(B) vibrating of objects
(C) objects vibrate
(D) the vibration of objects

10. Minnesota, _______, actually has 12,034 lakes.


42
(A) the Land of 10,000 lakes
(B) it is the Land of 10,000 lakes
(C) its Land of 10,000 lakes
(D) which a Land of 10,000 lakes

11. The lion and tiger are_______ of the cat family


(A) Larger than
(B) the largest
(C) largest
(D) as large as

12. Characteristically, the dulcimer features three or four strings_____along a fingersboard.


(A) stretched
(B) are stretched
(C) stretch them
(D) they are stretched

13. Diamond Head is what _____ of an extinct volcano.


(A) to remains
(B) remains
(C) remaining
(D) the remnants

14. Any member of a group of drugs____barbituric acid is a barbiturate.


(A) the derivation of
(B) is derived from
(C) derived from
(D) derives from

15. In 1890, Henry VIII ______ that people born on February 29 would celebrate their birthdays on
February 28 in non-leap years.
(A) rules
(B) ruled
(C) has ruled
(D) had ruled

16. Areas of rock may shifts up or down on each side of a fault


A B C D
17. When someone take a depressant, the level of activity in the central nervous system is lowered.
A B C D
18. The world’s largest deposits of asbestos have be located in the Appalachian Mountains.
A B C D
19. Both Thomas Jefferson or John Adams died on the very same day, July 4, 1826.
A B C D
20. Most tundra plant are mosses and lichens that hug ground.
A B C D
21. Uranium atoms in a nuclear reaction is split into smaller atoms to produce heat.
A B C D
22. Virginia Dare, the first child born to English parents in the new World, was named after the
A B C
Virginia colony where her was born.
D
23. The apple tree appears in the mythology, traditions, historical, and archeology of the most ancient
A B C D
nations.
24. The first horse-drawn chariost were introduce around 2500 B.C.
A B C D

25. In 1917, Clyde Cessna designed an innovative monoplane, and later the .Aircraft company was
43
A B C
started by he.
D
26. Not only the Estruscan alphabet and the Latin alphabet are descendants of the Greek alphabet.
A B C D

27. All of the oceans of the world is home to members of the dolphin family.
A B C D
28. The Earth rotates on its axis and follows an elliptically orbit around the Sun.
A B C D
29. Theodore Roosevelt was the first precident to ride in a car, fly in an airplane, and submerging in a
A B C
submarine.
D
30. The carbonation of water and soft drinks were one of the first uses found for gaseous carbon
A B C D
dioxide.
31. After an Apple is cut, chemicals inside her combine with oxygen to form a brown coat.
A B C D
32. Cribbage is a card game in which points are tally by moving pegs on a board.
A B C D
33. Most of the birds living in desert regions inhabits the fringers of the desert.
A B C D
34. The U.S Constitution, in Article 1, Section 2, provided for a census of the population every
A B C
decades.
D
35. The constellation Aquarius was associated with the rainy seasons by a large amount of ancient
A B C D
civilizations.
36. Only a small percentage of mined diamonds are actually suitably for use as gemstones.
A B C D
37. A turkey that Abraham Lincoln had save from Thanksgiving dinner became a beloved family pet.
A B C D
38. The game of dominoes is played with flat, oblong blocks identified by the number of dots on theirs
A B C D
faces.

39. Sodium hydroxide and other alkali solutions work effective as degreasing agents.
A B C D
40. The flintlock pistol was popular as aweapon in the seventeenth century because they could be
A B C
fired one-handed by a rider on horseback.
D

44
Practice Test Reading

Questions 1 – 10

Precipitation, commonly referred to as rainfall, is a measure of the quantity of water in the form
of either rain, hail, or snow which reaches the ground. The average annual precipitation over the
whole of the United States is thirty-six inches. It should be understood however, that a foot of snow
is not equal to a foot of precipitation. A general formula for computing the precipitation of snowfall is
(5) that ten inches of snow is equal to one inch of precipitation. In New York State, for example, twenty
inches of snow in one year would be recorded as only two inches of precipitation. Forty inches of
rain would be recorded as forty inches of precipitation. The total annual precipitation would be
recorded as forty-two inches.
The amount of precipitation is a combined result of several factors, including location, altitude,
(10) proximity to the sea, and the direction of prevailing winds. Most of the precipitation in the United
States is brought originally by prevailing winds from the Pacific Ocean, the Gulf of Mexico, the
Atlantic Ocean, and the Great Lakes. Because these prevailing winds generally come from the
west, the Pacific coast receives more annual precipitation than the Atlantic Coast. Along the Pacific
Coat itself, however, altitude causes some diversity in rainfall. The mountain ranges of the United
(15) States, especially the Rocky Mountains Range and the Appalachian Mountain Range, influence the
amount of precipitation in their areas. East of the Rocky Mountains, the annual precipitation
decreases substantially from that west of the Rocky Mountains. The precipitation north of the
Appalachian Mountains is about 40 percent less than that south of the Appalachian Mountains.

1. What does this passage mainly 6. The phrase “proximity to” in line 10 is
discuss? closest in meaning to?
(A) Precipitation (A) communication with
(B) Snowfall
(C) New York State (B) dependence on
(D) A general formula (C) nearness to
(D) similarly to
2. Which of the following is another word
that is often used in place of 7. Where is the annual precipitation
precipitation? highest?
(A) Humidity (A) The Atlantic Ocean
(B) Wetness (B) The Great Lakes
(C) Rainfall
(D) Rain-snow (C) The Gulf of Mexico
(D) The Pacific Ocean
3. The term precipitation includes
(A) only rainfall 8. Which of the following was NOT
(B) rain, hail and snow mentioned as a factor in determining
(C) rain, snow and humidity the amount of precipitation that an
(D) rain, hail and humidity area will receive?
4. What is the average annual rainfall in (A) Mountains
inches in the United States? (B) Latitude
(A) Thirty-six inches (C) The sea
(B) Thirty-eight inches (D) Wind
(C) Forty inches
(D) Forty-two inches 9. The word “substantially” in line 17
could best be replaced by
5. If a state has 40 inches of snow in a (A) fundamentally
year, by how much does this increase (B) slightly
the annual precipitation? (C) completely
(A) By two feet (D) apparently
(B) By four inches
(C) By four feet 10. The word “that” in line 18 refers to
(D) By 40 inches (A) decreases
(B) precipitation
(C) areas
(D) mountain range

45
Questions 11 – 20

Course numbers are an indication of which courses are open to various categories of students
at the university. Undergraduate courses with the numbers 100 or 200 are generally introductory
courses appropriate for freshmen or sophomores, whereas courses with the number 300 or 400
often have prerequisites and are open to juniors and seniors only. Courses with the numbers 800
(5) above are open only to graduate students. Certain graduate courses, generally those devoted to
introductory material, are numbered 400 for undergraduate students who qualify to take them and
600 for graduate students. Courses designed for students seeking a professional degree carry a
500 number for undergraduate students and a 700 number for graduate students. Courses
numbered 99 or below are special interest courses that do not carry academic credit. If students
(10) elect to take a special interest course, it will not count toward the number of hours needed to
complete graduation requirements.
A full-time undergraduate student is expected to take courses that total twelve to eighteen
credit hours. A full time graduate student is expected to take courses that total ten to sixteen credit
hours. Students holding assistantships are expected to enroll for proportionately fewer hours, a part
(15) time graduate student may register for a minimum of three credit hours. An overload, that is, more
than maximum number of hours, may be taken with the approval of an academic advisor. To
register for an overload, students must submit the appropriate approval form when registering.
Overloads above 24 hours will not be approved under any circumstances.

11. Where would this passage most likely 16. If an undergraduate student uses the
be found? number 520 to register for an
(A) In a syllabus accounting course, what number would
(B) In a college catalog a graduate student probably use to
(C) In an undergraduate course register for the same course?
(D) In an graduate course
(A) Accounting 520
(B) Accounting 620
12. What is the purpose of the passage?
(C) Accounting 720
(A) To inform
(D) Accounting 820
(B) To persuade
(C) To criticize
17. How is a student who registers for
(D) To apologize
eight credit hours classified?
(A) Full time student
13. The word “prerequisites” in line 4 is
(B) Graduate student
closest in meaning to
(C) Part time graduate student
(A) courses required before enrolling
(D) Non degree student
(B) courses needed for graduation
(C) courses that include additional
18. Which of the following courses would
charges
not be included in the list of courses for
(D) courses that do not carry
graduation?
academic credit
(A) English 90
(B) English 100
14. The word “those” in line 5 refers to
(C) English 300
(A) graduate students
(D) English 400
(B) graduate courses
(C) introductory courses
19. A graduate students may NOT
(D) course numbers
(A) enroll in a course numbered 610
(B) register for only one one-hour
15. Which classification of students would
course
be eligible to enroll in mechanical
(C) register for courses if he has an
Engineering 850?
assistantship
(A) A graduate student
(D) enroll in an introductory course
(B) A part time student
(C) A full time student
20. The phrase “under any circumstances” in
(D) An undergraduate student
lines 18 is closest in meaning to
(A) without cause
(B) without permission
(C) without exception
(D) without a good reason

46
Questions 21 – 30

The temperature of the Sun is over 5,000 degrees Fahrenheit at the surface, but it rises
to perhaps more than 16 million degrees at the center. The sun is so much hotter than the Earth
that matter can exist only as a gas, except at the core. In the core of the Sun, the pressures are
so great against the gases that, despite the high temperature, there may be a small solid core.
(5) However, no one really knows, since the center of the Sun can never be directly observed.
Solar astronomers do know that the Sun is divided into five layers or zones. Starting at
the outside and going down into the Sun, the zones are the corona, chromospheres,
photosphere, convection zone, and finally the core. The first three zones are regarded as the
Sun’s Atmosphere. But since the Sun has no solid surface, it is hard to tell where the
(10) atmosphere ends and the main body of the Sun begins.
The Sun’s outermost layer begins about 10,000 miles above the visible surface and goes
outward for millions of miles. This is the only part of the Sun that can be seen during an eclipse
such as the one in February 1979. At any other time, the corona can be seen only when special
instruments are used on cameras and telescopes to shut out the glare of the Sun’s rays.
(15) The corona is a brilliant, pearly white, filmy light, about as bright as the full moon. Its
beautiful rays are a sensational sight during an eclipse. The corona’s rays flash out in a brilliant
fan that has wispy spike like rays near the Sun’s north and south poles. The corona is thickest
at the Sun’s equator.
The corona rays are made up of gases streaming outward at tremendous speeds and
(20) reaching a temperature of more than 2 million degrees Fahrenheit. The rays of gas thin out as
they reach the space around the planets. By the time the Sun’s corona rays reach the Earth,
they are weak and invisible.

21. The word “great” in line 4 is closest in 26. It can be inferred from the passage
meaning to that a clear view of the Sun’s outer
(A) dangerous layer is usually prevented by
(B) unknown (A) the Sun’s rays
(C) variable (B) an eclipse
(D) strong (C) lack of light
(D) the great distance
22. With what topic is the second
paragraph mainly concerned? 27. The word ‘sensational” in line 16 is
(A) How the Sun evolved closest in meaning to
(B) The structure of the Sun (A) spectacular
(C) Why scientists study the Sun (B) predictable
(D) The distance of the Sun from the (C) bizarre
planets (D) constant

23. All of following are parts of the Sun’s 28. According to the passage as the
atmosphere EXCEPT the corona rays reach the planets, they
(A) corona become
(B) chromospheres (A) hotter
(C) photosphere (B) clearer
(D) core (C) thinner
(D) stronger
24. The word “one” in line 13 refers to
(A) the Sun 29. The paragraphs following the passage
(B) the corona most likely discuss
(C) an eclipse (A) The remaining layers of the Sun
(D) the surface (B) The evolution of the Sun
(C) The eclipse of February 1979
25. The purpose of the special instruments
(D) The scientists studying astronomy
mentioned in line 14 is to
(A) magnify the image of the Sun
30. Where in the passage does the author
(B) block out the Sun’s intense light
compare the light of the Sun’s
(C) measure the amount of energy
outermost layer to that of another
emitted by the Sun
astronomical body?
(D) photograph the Sun
(A) Lines 2 – 3
(B) Lines 9 – 10
(C) Line 17 – 18
(D) Lines 22 – 23

47
Questions 31 – 40

The Acacia is a genus of trees and shrubs of the Mimosa family. Although nearly five
hundred species of Acacia have been identified, only about a dozen of the three hundred
Australian varieties grow well in the southern United States, and these, only three are
flowering. The Bailey Acacia has fernlike silver leaves and small, fragrant flowers arranged in
(5) rounded clusters. The Silver Wattle, although very similar to the Bailey Acacia, grows twice as
high. The Sydney Golden Wattle is squat and bushy with broad, flat leaves and sharp spine
twigs. Named for its bright, yellow flowers, the Golden Wattle is the most showy and fragrant
of the Acacias. Another variety, the Black Acacia or Blackwood, has dark green leaves and
obtrusive blossoms. Besides being a popular tree for ornamental purposes, the Black Acacia
(10) is valuable for its dark wood, which is used in making cabinets and furniture, including highly
prized pianos.
The Acacia’s unusual custom of blossoming in February has been commonly attributed to
its Australian origins. In the Southern Hemisphere, of course, the seasons are reversed, and
February, which is wintertime in the United States, is summertime in Australia. Actually,
(15) however, the pale, yellow blossoms appear in August in Australia. Whether growing in the
Northern or Southern Hemisphere, the Acacia will bloom in winter.

31. With which of the following topics is the 36. The word “showy” in line 7 could best
passage primarily concerned? be replaced by
(A) The Black Acacia (A) strange
(B) Characteristics and varieties of (B) elaborate
(C) huge
the acacia (D) fragile
(C) Australian varieties of the acacia
(D) The use of Acacia wood in 37. Which of the following Acacia has the
ornamental furniture least colorful blossoms?
(A) Bailey Acacia
32. How many Australian varieties of (B) Sydney Golden Wattle
Acacia grow well in the southern (C) Silver Wattle
United States? (D) Black Acacia
(A) Five hundred
38. Which of the following would most
(B) Three hundred probably be made from a Black Acacia
(C) Twelve tree?
(D) Three (A) A flower arrangement
(B) A table
33. The word “these” in line 3 refers to (C) A pie
(A) United States (D) Paper
(B) Australian varieties
(C) Acacia species 39. The phrase “highly prized” in lines 11
(D) Trees and shrubs is closest meaning to
(A) valuable
34. According to this passage, the Silver (B) unique
Wattle (C) stylish
(A) is squat and bushy (D) attractive
(B) has unobtrusive blossoms
(C) is taller than the Bailey Acacia 40. When do Acacia trees bloom in
(D) is used for making furniture Australia?
(A) February
35. In line 6, the word “flat” the most nearly (B) Winter
means (C) Before August
(A) smooth (D) Spring
(B) pretty
(C) pointed
(D) short

48
Questions 41 – 50

In 1626, Peter Minuit, governor of Dutch settlements in North America known as New
Amsterdam, negotiated with Canaries Indian chiefs for the purchase of Manhattan Island for
merchandise valued at sixty guilders or about $24.12. He purchased the island for the Dutch
West India Company.
(5) The next year, Fort Amsterdam was built by the company at the extreme southern tip of
the island. Because attempts to encourage Dutch immigration were not immediately
successful, offers, generous by the standards of the area, were extended throughout Europe.
Consequently, the settlement became the most heterogeneous of the North American
colonies. By 1637, the fort had expanded into the village of New Amsterdam, and other small
(10) communities had grown up around it, including New Harlem and Stuyvesant’s Bourwery, and
New Amsterdam began to prosper, developing characteristics of religious and linguistic
tolerance unusual for the times. By 1643, it was reported that eighteen different language were
heard in New Amsterdam alone.
Among the multilingual settlers was a large group of English colonies from Connecticut
(15) and Massachusetts who supported the English King’s claim to all of New Netherlands set out
in charter that gave the territory to his brother James, the Duke of York. In 1664, when the
English sent a formidable fleet of warship into the New Amsterdam harbor, Dutch governor
Peter Stuyvesant surrendered without resistance.
When the English acquired the island, the village of New Amsterdam was renamed New
(20) York in honor of Duke. By the onset of the revolution, New York City was already a bustling
commercial center. After the war, it was selected as the first capital of the United States.
Although the government was eventually moved, first to Philadelphia and then to Washington
D.C., New York City has remained the unofficial commercial capital.
During the 1690’s, New York became a haven for pirates who conspired with leading
(25) merchants to exchange many agricultural products for English manufactured goods. In
addition, trade with the West Indies prospered. Three centuries after his initial trade with the
Indians, Minuit’s tiny investment was worth more than seven billion dollars.

41. Which of the following would be the before the purchase spoke many
best title for this passage? languages
(A) A history of New York City
(B) An account of the Dutch Colonies 46. The word “formidable” in line 18 is
(C) A Biography of Peter Minuit closest in meaning to
(D) The first Capital of the United (A) powerful
States (B) modern
(C) expensive
42. What did the Indians receive in (D) unexpected
exchange for their island?
(A) Sixty Dutch guilders 47. The name of New Amsterdam was
(B) $ 24.12 U.S. changed
(C) Goods and supplies (A) to avoid a war with England
(D) Land in New Amsterdam (B) to honor the Duke of York
(C) to attract more English colonies
43. Where was New Amsterdam located? from Connecticut and
(A) In Holland Massachusetts
(B) In North America (D) to encourage trade during the
(C) On the island of Manhattan 1690s
(D) In India
48. The word “it” in line 21 refers to
44. The word “heterogeneous” in line 8 (A) Revolution
could best be replaced by (B) New York City
(A) liberal
(C) The island
(B) renowned
(C) diverse (D) The first capital
(D) prosperous
49. Which city was the first capital of the
45. Why were so many languages spoken new United States?
in New Amsterdam? (A) New Amsterdam
(A) Because the Dutch West India (B) New York
Company was owned by England (C) Philadelphia
(B) Because the Dutch West India (D) Washington
Company allowed freedom of
speech 50. On what date was Manhattan valued at
(C) Because the Dutch West India $7 billion?
Company recruited settlers from (A) 1626
many different countries in Europe (B) 1726
(D) Because the Indians who lived (C) 1656
there

49

You might also like